Uniform Convergence of Complex Series

  • Thread starter Thread starter futurebird
  • Start date Start date
  • Tags Tags
    Complex Series
Click For Summary
The discussion centers on the uniform convergence of two series involving complex variables. The first series, ∑z^n, converges uniformly for |z|<R with R<1, and the solution shows that the nth partial sums approach z/(1-z) as n approaches infinity. For the second series, the sequence of functions f_n=z^n converges uniformly to zero, indicating that it is Cauchy as well. There is some confusion regarding the interpretation of uniform convergence in the context of the second question, but the responses clarify that both series exhibit uniform convergence. Overall, the initial analysis of the first series is deemed correct, while the second question's approach is validated.
futurebird
Messages
270
Reaction score
0
This is a question from a homework assignment that I turned in today and it's driving me nuts because I don't know if I did it correctly or not. First of all we have these two questions that seem almost identical:

Question 1
Show that the following series converges uniformly in the given region.
\displaystyle\sum_{n=1}^{\infty}z^{n} for 0\le|z|&lt;R and R&lt;1Question 2
Show that the sequence:
\displaystyle\left\{x^{n}\right\}^{\infty}_{n=1}
converges uniformly inside 0\le|z|&lt;R with R&lt;1.​

How I did question 1:

\displaystyle\sum_{n=1}^{\infty}z^{n}= z + z^{2} + z^{3} + \cdots + z^{n} + \cdots

Call the nth partial sum of the series S_{n}:

S_{n}= z + z^{2} + z^{3} + \cdots + z^{n}

zS_{n}= z^{2} + z^{3} + \cdots + z^{n} + z^{n+1}

S_{n}=\frac{z(1 - z^{n})}{1-z}

Now,

\displaystyle\sum_{n=1}^{\infty}z^{n}= \displaystyle\lim_{n\to\infty}S_{n}=\displaystyle\lim_{n\to\infty}\frac{z(1 - z^{n})}{1-z}

=\frac{z}{1-z} because z^{n}\longrightarrow 0 for |z|&lt;1

Now we consider:

\left|\frac{z}{1-z} -\frac{z(1 - z^{n})}{1-z}\right| = \frac{|z|^{n}}{|1-z|}&lt; \frac{R^{n}}{1-R}

with \epsilon&gt;0 we choose N so that N depends on R and \epsilon but not on on z, so that when n&gt;N_{R,\epsilon}

\frac{R^{n}}{1-R}&lt; \epsilon

Because we can choose such an N the series converges uniformly.

For question 2 I was confused, If
\displaystyle\sum_{n=1}^{\infty}z^{n}
converges then the sequence \displaystyle\left\{x^{n}\right\}^{\infty}_{n=1}

must be cauchy, right? So as n\longrightarrow \infty the nth term of the series goes to zero. So I thought I'd need to show that there was a N, not dependent on z such that the nth term would be less than \epsilon.

I found:

N = \frac{log \epsilon}{log R} + 1

Is that good enough to show uniform convergence?
 
Last edited:
Physics news on Phys.org
I think you are right. But your language is somewhat confusing. "Uniform convergence" is a term to apply to a sequence of functions. In the first case, the sequence of functions is the partial sums of your series, and yes, they converge uniformly to z/(1-z) over the domain |z|<=R<1. In the second case the sequence of functions is f_n=z^n. And they converge uniformly to zero. And, yeah, the second sequence of function are differences of the first sequence of functions, so if the first sequence of functions converges, the second sequence converges to zero. I'm a little confused about what the question is.
 
I just didn't know if ai was waaaaaay off base or not about what the 2nd quetions was asking-- and I wanted to know if I did the first one right.

And I like typing things in latex... that's all! :smile:

Thanks for responding.
 
You could install a latex compiler and typeset mathematics equations to your hearts content in the privacy of your own home. Anyway, it looks pretty much ok to me.
 
Question: A clock's minute hand has length 4 and its hour hand has length 3. What is the distance between the tips at the moment when it is increasing most rapidly?(Putnam Exam Question) Answer: Making assumption that both the hands moves at constant angular velocities, the answer is ## \sqrt{7} .## But don't you think this assumption is somewhat doubtful and wrong?

Similar threads

  • · Replies 3 ·
Replies
3
Views
2K
  • · Replies 7 ·
Replies
7
Views
2K
  • · Replies 6 ·
Replies
6
Views
2K
  • · Replies 3 ·
Replies
3
Views
1K
  • · Replies 3 ·
Replies
3
Views
1K
  • · Replies 9 ·
Replies
9
Views
2K
  • · Replies 2 ·
Replies
2
Views
2K
  • · Replies 2 ·
Replies
2
Views
2K
  • · Replies 13 ·
Replies
13
Views
2K
Replies
12
Views
3K